0
7.8kviews
Determine the 8-point DFT of the sequence x(n)={1,1,1,1,1,1,0,0}
1 Answer
0
1.1kviews

Solution:

$ X(\mathrm{k})=\sum_{n=0}^{N-1} x(\mathrm{n}) \mathrm{e}^{-j 2 \pi k n / N} k=0,1, \ldots N-1 $

For $\mathrm{N}=8$

$ X(\mathrm{k})=\sum_{n=0}^7 x(\mathrm{n}) \mathrm{e}^{-j \pi k n / 4} k=0,1,2 \ldots . . N-1\\ $

For k=0

$ \begin{aligned} X(0) & =\sum_{n=0}^7 x(n) \\\\ \mathrm{X}(0) & =\mathrm{x}(0)+\mathrm{x}(1)+\mathrm{x}(2)+\mathrm{x}(3)+\mathrm{x}(4)+\mathrm{x}(5)+\mathrm{x}(6)+\mathrm{x}(7) \\\\ & =1+1+1+1+1+1+0+0 \\\\ & =6 \end{aligned} $

For $\mathrm{k}=1$

$ \begin{aligned} X(1) & =\sum_{n=0}^7 x(n) \mathrm{e}^{-j \pi n / 4} \\\\ \mathrm{X}(1) & =\mathrm{x}(0)+\mathrm{x}(1) \mathrm{e}^{-\mathrm{j} \pi / 4}+\mathrm{x}(2) \mathrm{e}^{-\mathrm{j} \pi / 2}+\mathrm{x}(3) \mathrm{e}^{-\mathrm{j} 3 \pi / 4}+\mathrm{x}(4) \mathrm{e}^{-\mathrm{j} \pi}+\mathrm{x}(5) \mathrm{e}^{-\mathrm{j} 5 \pi / 4}+\mathrm{x}(6) \mathrm{e}^{-\mathrm{j} 3 \pi / 2}+\mathrm{x}(7) \mathrm{e}^{-\mathrm{j} 7 \pi / 4} \\\\ & =1+0.707-\mathrm{j} 0.707-\mathrm{j}-0.707-\mathrm{j} 0.707-1-0.707+\mathrm{j} 0.707 \quad=-0.707-\mathrm{j} 1.707\\ \end{aligned} $

For $\mathrm{k}=2$

$ \begin{aligned} X(2) & =\sum_{n=0}^7 x(n) e^{-j \pi n / 2} \\\\ X(2) & =x(0)+\mathrm{x}(1) \mathrm{e}^{-\mathrm{j} \pi / 2}+\mathrm{x}(2) \mathrm{e}^{-\mathrm{j} \pi}+\mathrm{x}(3) \mathrm{e}^{-\mathrm{j} 3 \pi / 2}+\mathrm{x}(4) \mathrm{e}^{-\mathrm{j} 2 \pi}+\mathrm{x}(5) \mathrm{e}^{-\mathrm{j} 5 \pi / 2}+\mathrm{x}(6) \mathrm{e}^{-\mathrm{j} 3 \pi}+\mathrm{x}(7) \mathrm{e}^{-\mathrm{j} 7 \pi / 2} \\\\ & =1-\mathrm{j}-1+\mathrm{j}+1-\mathrm{j} \\\\ & =1-\mathrm{j}\\ \end{aligned} $

For $\mathrm{k}=3$

$ \begin{aligned} X(3) & =\sum_{n=0}^7 x(n) e^{-j 3 \pi n / 4} \\\\ X(3) & =x(0)+x(1) e^{-j 3 \pi / 4}+\mathrm{x}(2) \mathrm{e}^{-\mathrm{j} \pi / 2}+\mathrm{x}(3) \mathrm{e}^{-\mathrm{j} 9 \pi / 4}+\mathrm{x}(4) \mathrm{e}^{-\mathrm{j} 3 \pi}+\mathrm{x}(5) \mathrm{e}^{-\mathrm{j} 15 \pi / 4}+\mathrm{x}(6) \mathrm{e}^{-\mathrm{j} 9 / / 4}+\mathrm{x}(7) \mathrm{e}^{-\mathrm{j} 21 \pi / 4} \\\\ & =1-0.707-\mathrm{j} 0.707+\mathrm{j}+0.707-\mathrm{j} 0.707-1+0.707+\mathrm{j} 0.707 \\\\ & =0.707+\mathrm{j} 0.293\\ \end{aligned} $

For $\mathrm{k}=4$

$ \begin{aligned} X(4) & =\sum_{n=0}^7 x(n) \mathrm{e}^{-j \pi n}\\ \\ X(4) & =\mathrm{x}(0)+\mathrm{x}(1) \mathrm{e}^{-j \pi}+\mathrm{x}(2) \mathrm{e}^{-j \pi 2}+\mathrm{x}(3) \mathrm{e}^{-j \pi 3}+\mathrm{x}(4) \mathrm{e}^{-\mathrm{j} \pi 4}+\mathrm{x}(5) \mathrm{e}^{-\mathrm{j} \pi 5}+\mathrm{x}(6) \mathrm{e}^{-j \pi 6}+\mathrm{x}(7) \mathrm{e}^{-\mathrm{j} \pi 7} \\\\ & =1-1+1-1+1-1=0 \end{aligned}\\ $

For $\mathrm{k}=5$

$ \begin{aligned} X(5) & =\sum_{n=0}^7 x(n) e^{-j 5 \pi n / 4} \\\\ X(5) & =x(0)+\mathrm{x}(1) \mathrm{e}^{-\mathrm{j} 5 \pi / 4}+\mathrm{x}(2) \mathrm{e}^{-\mathrm{j} 5 \pi / 2}+\mathrm{x}(3) \mathrm{e}^{-\mathrm{j} 5 \pi n / 4}+\mathrm{x}(4) \mathrm{e}^{-\mathrm{j} 5 \pi}+\mathrm{x}(5) \mathrm{e}^{-\mathrm{j} 25 \pi / 4}+\mathrm{x}(6) \mathrm{e}^{-\mathrm{j} 15 \pi / 2}+\mathrm{x}(7) \mathrm{e}^{-\mathrm{j} 35 \pi / 4} \\\\ & =1-0.707+\mathrm{j} 0.707-\mathrm{j}+0.707+\mathrm{j} 0.707-1+0.707-\mathrm{j} 0.707 \\\\ & =0.707-\mathrm{j} 0.293\\ \end{aligned} $

For $\mathrm{k}=6$

$ \begin{aligned} X(6) & =\sum_{n=0}^7 x(n) e^{-j 3 \pi n / 2} \\\\ \mathrm{X}(6) & =\mathrm{x}(0)+\mathrm{x}(1) \mathrm{e}^{-\mathrm{j} 3 \pi / 2}+\mathrm{x}(2) \mathrm{e}^{-\mathrm{j} 3 \pi}+\mathrm{x}(3) \mathrm{e}^{-\mathrm{j} 9 \pi / 2}+\mathrm{x}(4) \mathrm{e}^{-\mathrm{j} 6 \pi}+\mathrm{x}(5) \mathrm{e}^{-\mathrm{j} 15 \pi}+\mathrm{x}(6) \mathrm{e}^{-\mathrm{j} 9 \pi}+\mathrm{x}(7) \mathrm{e}^{-\mathrm{j} 21 \pi / 2} \\\\ & =1+\mathrm{j}-1-\mathrm{j}+1+\mathrm{j} \Rightarrow \quad=1+\mathrm{j}\\ \end{aligned} $

For $\mathrm{k}=7$

$ X(7)=\sum_{n=0}^7 x(\mathrm{n}) e^{-j 7 \pi n / 4}\\ $

$ \begin{aligned} \mathrm{X}(7) & =\mathrm{x}(0)+\mathrm{x}(1) \mathrm{e}^{-\mathrm{j} 7 \pi / 4}+\mathrm{x}(2) \mathrm{e}^{-\mathrm{j} 7 \pi / 2}+\mathrm{x}(3) \mathrm{e}^{-\mathrm{j} 21 \pi / 4}+\mathrm{x}(4) \mathrm{e}^{-\mathrm{j} 7 \pi}+\mathrm{x}(5) \mathrm{e}^{-\mathrm{j} 35 \pi / 4}+\mathrm{x}(6) \mathrm{e}^{-\mathrm{j} 21 \pi / 2}+\mathrm{x}(7) \mathrm{e}^{-\mathrm{j} 49 \pi / 4} \\\\ & =1+0.707+\mathrm{j} 0.707+\mathrm{j}-0.707+\mathrm{j} 0.707-1-0.707-\mathrm{j} 0.707 \\\\ = & -0.707+\mathrm{j} 1.707 \\\\ & \mathrm{X}(\mathrm{K})=\{6,0.707-\mathrm{j} 1.707,1-\mathrm{j}, 0.707+\mathrm{j} 0.293,0,0.707-\mathrm{j} 0.293,1+\mathrm{j},-0.707+\mathrm{j} 1.707\}\\ \end{aligned} $

Please log in to add an answer.